Mathcenter Forum  

Go Back   Mathcenter Forum > คณิตศาสตร์โอลิมปิก และอุดมศึกษา > ข้อสอบโอลิมปิก
สมัครสมาชิก คู่มือการใช้ รายชื่อสมาชิก ปฏิทิน ข้อความวันนี้

ตั้งหัวข้อใหม่ Reply
 
เครื่องมือของหัวข้อ ค้นหาในหัวข้อนี้
  #376  
Old 02 พฤษภาคม 2012, 19:18
passer-by passer-by ไม่อยู่ในระบบ
ผู้พิทักษ์กฎทั่วไป
 
วันที่สมัครสมาชิก: 11 เมษายน 2005
ข้อความ: 1,442
passer-by is on a distinguished road
Default

อ้างอิง:
ข้อความเดิมเขียนโดยคุณ Beatmania View Post
$(1)+(2) \rightarrow 2r sin ( \theta _A+ \theta _B)cos( \theta _A- \theta _B) = -2 sin ( \theta _C) cos ( \theta _A- \theta _B)$
$r sin ( \theta _A+ \theta _B)= - sin ( \theta _C) (4)$
$(1)-(2) \rightarrow 2r cos ( \theta _A+ \theta _B) sin ( \theta _A- \theta _B) = -2 cos( \theta _C) sin ( \theta _B- \theta _A)$
$r cos ( \theta _A+ \theta _B)= cos( \theta _C) (5)$
$(4)^2+(5)^2 \rightarrow r^2=1$
แต่ $r>0$ ดังนั้น $r=1$
ที่เหลือก็น่าจะต่อได้นะครับ ^^
กรณี $cos ( \theta _A- \theta _B) = 0$ ยังไม่ได้เช็คครับ

แล้วก็ จาก $ r=1 $ ทำไม $ abc =1$ ล่ะครับ

--------------------------------------------------------------------------

Note :ข้อนี้มีอีกวิธีที่ไม่ใช้ cis ครับ
__________________
เกษียณตัวเอง ปลายมิถุนายน 2557 แต่จะกลับมาเป็นครั้งคราว
ตอบพร้อมอ้างอิงข้อความนี้
  #377  
Old 03 พฤษภาคม 2012, 23:04
Beatmania's Avatar
Beatmania Beatmania ไม่อยู่ในระบบ
ลมปราณคุ้มครองร่าง
 
วันที่สมัครสมาชิก: 10 พฤษภาคม 2011
ข้อความ: 279
Beatmania is on a distinguished road
Default

อ้างอิง:
ข้อความเดิมเขียนโดยคุณ passer-by View Post
กรณี $cos ( \theta _A- \theta _B) = 0$ ยังไม่ได้เช็คครับ

Note :ข้อนี้มีอีกวิธีที่ไม่ใช้ cis ครับ
ขอติดกรณีไว้ก่อนหน่ะครับ ขอโทษด้วย

จากที่ $r=1$ และเงื่อนไข $(4)$ และ $(5)$ จะได้ว่า $\theta _A+\theta _B$ และ $\theta _C$

เป็นมุมที่สะท้อนซึ่งกันและกันบนแกน x เมื่อเขียน a b c ในรูปพิกัดเชิงขั้ว (ที่จริงวาดรูปไว้ในกระดาษอ่ะครับ)

กล่าวคือ $\theta _A+\theta _B=2\pi -\theta _C$

$\theta _A+\theta _B+\theta _C=2\pi $

$abc = cis( \theta _A+\theta _B+\theta _C )= cis 2\pi = 1$ ครับ

เพิ่มโจทย์อสมการซักข้อดีกว่าครับ

ถ้า $xyz=1$ และ $x,y,z \in \unicode{8477} $ แล้วจงแสดงว่า

$\frac{x^2}{(x-1)^2} +\frac{y^2}{(y-1)^2} +\frac{z^2}{(z-1)^2} \geqslant 1$

$(จุดที่โจทย์ข้อนี้ต้องการจริงๆ คือ แสดงว่า มี x,y,z ที่เป็นจำนวนตรรกยะเป็นอนันต์ที่ทำให้อสมการเป็นสมการครับ [IMO 2008 / 2])$

แถมอีกนิดครับ ข้อนี้ผมเห็นเฉลย 2 วิธี มีวิธีนึงใช้ C.R.T ครับ

จงแสดงว่า ทุกจำนวนเต็มบวก $n$ มีจำนวนนับ $n$ จำนวนที่ถัดกัน โดยไม่มีจำนวนใดอยู่ในรูป $p^k$ สำหรับจำนวนเฉพาะ p บางจำนวน

(อดีต IMO ครับ จำไม่ได้ว่าปีไหน)

04 พฤษภาคม 2012 23:03 : ข้อความนี้ถูกแก้ไขแล้ว 8 ครั้ง, ครั้งล่าสุดโดยคุณ Beatmania
เหตุผล: พิมพ์ผิดเยอะมากกกกกกกกกกกกกกกกกกกกกกกกกกกกกกกกกกกกกกกกก
ตอบพร้อมอ้างอิงข้อความนี้
  #378  
Old 03 พฤษภาคม 2012, 23:41
win1234 win1234 ไม่อยู่ในระบบ
จอมยุทธ์หน้าใหม่
 
วันที่สมัครสมาชิก: 24 ธันวาคม 2010
ข้อความ: 87
win1234 is on a distinguished road
Default

ตอนนี้อยากทำ GI ครับ เลยขอเอาโจทย์ง่ายๆมาลงดูเล่นๆก่อน

Let ABC be an acute angled triangle and let H be its ortho centre.Let $h_{max}$ denote the

largest altitude of the triangle ABC.

Prove that $AH + BH + CH \leqslant2h_{max}$.

แล้วก็อีกข้อนึงครับแถมๆ

Find all natural numbers $n\geqslant1$ such that $n^2$ does not divide $(n-2)!$.
ตอบพร้อมอ้างอิงข้อความนี้
  #379  
Old 04 พฤษภาคม 2012, 21:31
~ArT_Ty~'s Avatar
~ArT_Ty~ ~ArT_Ty~ ไม่อยู่ในระบบ
ลมปราณไร้สภาพ
 
วันที่สมัครสมาชิก: 03 กรกฎาคม 2010
ข้อความ: 1,081
~ArT_Ty~ is on a distinguished road
Default

มาเสริมอีกข้อ เป็นตัวเลือกฮะ

มีฟังก์ชัน $f:\mathbb{N} \rightarrow \mathbb{N} $ ที่สอดคล้องกับ $f(f(n))=2n$ ทุก $n$ ที่เป็นจำนวนนับหรือไม่

อาจจะเคยผ่านๆตามาบ้างแล้วนะครับสำหรับข้อนี้
__________________
...สีชมพูจะไม่จางด้วยเหงื่อ แต่จะจางด้วยนํ้าลาย...
ตอบพร้อมอ้างอิงข้อความนี้
  #380  
Old 05 พฤษภาคม 2012, 00:01
win1234 win1234 ไม่อยู่ในระบบ
จอมยุทธ์หน้าใหม่
 
วันที่สมัครสมาชิก: 24 ธันวาคม 2010
ข้อความ: 87
win1234 is on a distinguished road
Default

มาต่อ GI อีกข้อครับ

กำหนดให้ a,b,c เป็นด้านของสามเหลี่ยมใดๆ และ R เป็นรัศมีวงกลมล้อมรอบสามเหลี่ยม ABC

จงพิสูจน์ว่า $$\frac{a^2}{b+c-a}+\frac{b^2}{c+a-b}+\frac{c^2}{a+b-c}\geqslant3\sqrt{3}R $$

ปล.#378 นั้นไม่มีอะไรยากจริงๆครับ และผมขอเพิ่มนิดนึงเป็นพิสูจน์ว่า $2h_{min}\leqslant AH+BH+CH\leqslant 2h_{max}$
ตอบพร้อมอ้างอิงข้อความนี้
  #381  
Old 05 พฤษภาคม 2012, 07:17
จูกัดเหลียง's Avatar
จูกัดเหลียง จูกัดเหลียง ไม่อยู่ในระบบ
ลมปราณไร้สภาพ
 
วันที่สมัครสมาชิก: 21 กุมภาพันธ์ 2011
ข้อความ: 1,234
จูกัดเหลียง is on a distinguished road
Default

#377 มันจริงสำหรับทุก $x,y,z\in R$ เหรอครับ เพราะถ้าลองเเทน $x=-1,y=-1/2,z=2$ ก็ขัดเเย้งเเล้ว
ผมว่าน่าจะเป็นจำนวนจริงบวกนะครับ = =
ปล.ตัวเลือกเยอะก็จริงเเต่ก็ยากหมดเลย 555+
#380
__________________
Vouloir c'est pouvoir

05 พฤษภาคม 2012 12:29 : ข้อความนี้ถูกแก้ไขแล้ว 4 ครั้ง, ครั้งล่าสุดโดยคุณ จูกัดเหลียง
ตอบพร้อมอ้างอิงข้อความนี้
  #382  
Old 05 พฤษภาคม 2012, 08:59
Beatmania's Avatar
Beatmania Beatmania ไม่อยู่ในระบบ
ลมปราณคุ้มครองร่าง
 
วันที่สมัครสมาชิก: 10 พฤษภาคม 2011
ข้อความ: 279
Beatmania is on a distinguished road
Default

#381 ปรับเป็น 1 แล้วนะครับ

(ถ้าพิสูจน์ว่ามันมากกว่า 1 ได้ ส่วนที่เหลือมันก็คล้ายกับโจทย์ข้อนึงในกระทู้นี้เลยแหละครับ )
ตอบพร้อมอ้างอิงข้อความนี้
  #383  
Old 05 พฤษภาคม 2012, 10:18
~ArT_Ty~'s Avatar
~ArT_Ty~ ~ArT_Ty~ ไม่อยู่ในระบบ
ลมปราณไร้สภาพ
 
วันที่สมัครสมาชิก: 03 กรกฎาคม 2010
ข้อความ: 1,081
~ArT_Ty~ is on a distinguished road
Default

มีโจทย์มาเติมครับ ข้อนี้ไม่ยากมาก
Let ABC be a triangle with |AC | > |AB|. Let the X be the intersection of the
perpendicular bisector of BC and the internal angle bisector of A. Let P,Q be the
foot of the perpendicular from X on AB extended and AC . Let Z be the intersection
of PQ and BC. Find the ratio BZ/ZC
__________________
...สีชมพูจะไม่จางด้วยเหงื่อ แต่จะจางด้วยนํ้าลาย...
ตอบพร้อมอ้างอิงข้อความนี้
  #384  
Old 05 พฤษภาคม 2012, 12:21
จูกัดเหลียง's Avatar
จูกัดเหลียง จูกัดเหลียง ไม่อยู่ในระบบ
ลมปราณไร้สภาพ
 
วันที่สมัครสมาชิก: 21 กุมภาพันธ์ 2011
ข้อความ: 1,234
จูกัดเหลียง is on a distinguished road
Default

#382 ถ้าเป็น $1$ ก็ได้เเล้วนี่ครับ กระจายจัดรูปได้ว่าสิ่งที่ต้องพิสูจน์คือ $\dfrac{(q-3)^2}{(p-q)^2}\ge 0$
ซึ่งถ้า $x,y,z>0$ ก็ได้ว่ามันมากกว่าเท่ากับ $5$ จริงๆนั่นเเหละครับ เเต่ตอนนี้ผมไม่รู้ว่าสมกาจะเกิดเมื่อใด
ปล.อยากได้คำแปลด้วยครับ = =
__________________
Vouloir c'est pouvoir

05 พฤษภาคม 2012 12:27 : ข้อความนี้ถูกแก้ไขแล้ว 1 ครั้ง, ครั้งล่าสุดโดยคุณ จูกัดเหลียง
ตอบพร้อมอ้างอิงข้อความนี้
  #385  
Old 05 พฤษภาคม 2012, 16:40
~ArT_Ty~'s Avatar
~ArT_Ty~ ~ArT_Ty~ ไม่อยู่ในระบบ
ลมปราณไร้สภาพ
 
วันที่สมัครสมาชิก: 03 กรกฎาคม 2010
ข้อความ: 1,081
~ArT_Ty~ is on a distinguished road
Default

ข้อ GI ของคุณ win1234 นะครับ

Name:  Untitled.jpg
Views: 669
Size:  63.0 KB

ผม Assume ว่า $AB\leqslant AC\leqslant BC$ จะได้ว่าเส้นส่วนสูง $AD=h_{min}$ และ $CF=h_{max}$

ต่อ $AD,BE,CF$ ไปตัดเส้นรอบวงวงกลมล้อมรอบสามเหลี่ยมที่ $J,K,L$ ตามลำดับนะครับ

จะได้ว่า $AL=AH=AK$ และ $BL=BH=BJ$ และ $CJ=CH=CK$ (พิสูจน์เองว่าทำไม ^^ แต่สำคัญมากในนี้)

จาก Ptolemy จะได้ว่า $BJ\times AC+CJ\times AB=AJ\times BC\leqslant BH\times BC+CH\times BC $

$\therefore AJ\leqslant BH+CH$ ทำให้ $AH+BH+CH\geqslant AH+AJ = 2AD = 2h_{min}$

ส่วนอสมการอีกข้างก็ทำคล้ายๆกันครับ ก็จะได้ดังโจทย์ครับ

โดยที่อสมการจะเท่ากันก็ต่อเมื่อ $ABC$ เป็นสามเหลี่ยมด้านเท่าครับ Q.E.D.

ป.ล. เส้นผมบังภูเขาจริงๆครับ ^^"
__________________
...สีชมพูจะไม่จางด้วยเหงื่อ แต่จะจางด้วยนํ้าลาย...

05 พฤษภาคม 2012 16:40 : ข้อความนี้ถูกแก้ไขแล้ว 1 ครั้ง, ครั้งล่าสุดโดยคุณ ~ArT_Ty~
ตอบพร้อมอ้างอิงข้อความนี้
  #386  
Old 05 พฤษภาคม 2012, 18:49
DoubleA1412 DoubleA1412 ไม่อยู่ในระบบ
สมาชิกใหม่
 
วันที่สมัครสมาชิก: 20 เมษายน 2012
ข้อความ: 2
DoubleA1412 is on a distinguished road
Default

ขอเสนอวิธีการพิสูจน์ $AH+BH+CH≤2h_{max}$ อีกวิธีนะครับ
ให้ AD,BE และ CF เป็นส่วนสูงที่ลากจากมุมยอด A,B และ C ตามลำดับ ซึ่งตัดกันที่ H
สมมติให้ AD เป็นส่วนสูงที่มีความยาวมากที่สุด

พิจารณา
$\frac{AH}{AD}$ = $\frac{[AHC]}{[ADC]}$ = $\frac{[AHB]}{[ADB]}$ = 1 - $\frac{[CHB]}{[ABC]}$

ในทำนองเดียวกันจะได้ $\frac{BH}{BE}$ = 1 - $\frac{[AHC]}{[ABC]}$
และ $\frac{CH}{CF}$ = 1 - $\frac{[BHA]}{[ABC]}$

จะได้ว่า $\frac{AH}{AD}$ + $\frac{BH}{BE}$ + $\frac{CH}{CF}$ = 3 - $\frac{[ABC]}{[ABC]}$ = 3-1 = 2

จะเห็นว่า $\frac{AH}{AD}$ + $\frac{BH}{AD}$ + $\frac{CH}{AD}$ $\leqslant$ $\frac{AH}{AD}$ + $\frac{BH}{BE}$ + $\frac{CH}{CF}$ = 2

ดังนั้น $AH+BH+CH≤2AD$ นั่นคือ $AH+BH+CH≤2h_{max}$

ส่วนกรณีของ $2h_{min}$ สมมติให้ AD เป็นส่วนสูงที่สั้นที่สุด แล้วทำในทำนองเดียวกัน

ปล. ต้องขอโทษด้วยครับถ้าอ่านยากเพราะผมยังใช้ LaTex ไม่ค่อยคล่องครับ

05 พฤษภาคม 2012 19:06 : ข้อความนี้ถูกแก้ไขแล้ว 1 ครั้ง, ครั้งล่าสุดโดยคุณ DoubleA1412
ตอบพร้อมอ้างอิงข้อความนี้
  #387  
Old 05 พฤษภาคม 2012, 19:19
Thgx0312555's Avatar
Thgx0312555 Thgx0312555 ไม่อยู่ในระบบ
กระบี่ประสานใจ
 
วันที่สมัครสมาชิก: 12 สิงหาคม 2011
ข้อความ: 885
Thgx0312555 is on a distinguished road
Default

(#378)::
อ้างอิง:
ข้อความเดิมเขียนโดยคุณ win1234 View Post
Find all natural numbers $n\geqslant1$ such that $n^2$ does not divide $(n-2)!$.

#379
hint แบ่งกลุ่มจำนวนนับ เป็น {1,2,4,...},{3,6,12,...},...
__________________
----/---~Alice~ จงรับรู้ไว้ ชื่อแห่งสีสันหนึ่งเดียวที่แสดงผล
---/---- ~Blue~ นี่คือ สีแห่งความหลังอันกว้างใหญ่ของเว็บบอร์ดนี้

07 พฤษภาคม 2012 18:09 : ข้อความนี้ถูกแก้ไขแล้ว 2 ครั้ง, ครั้งล่าสุดโดยคุณ Thgx0312555
เหตุผล: พิมพ์ผิด-ลืมเช็คคำตอบครับ
ตอบพร้อมอ้างอิงข้อความนี้
  #388  
Old 05 พฤษภาคม 2012, 19:38
~ArT_Ty~'s Avatar
~ArT_Ty~ ~ArT_Ty~ ไม่อยู่ในระบบ
ลมปราณไร้สภาพ
 
วันที่สมัครสมาชิก: 03 กรกฎาคม 2010
ข้อความ: 1,081
~ArT_Ty~ is on a distinguished road
Default

อ้างอิง:
ข้อความเดิมเขียนโดยคุณ Thgx0312555 View Post
(#378)::

พิจารณา $n \geqslant 4$

จะพบว่า $\dfrac{n}{2} \geqslant n-2$ นั่นคือตัวประกอบทั้งหมดที่ไม่ใช่ตัวมันเองจะมีค่าน้อยกว่าหรือเท่ากับ n-2 ---(1)

ให้ $n = p_1^{i_1}p_2^{i_2}...p_k^{i_k}$ พิจารณา $p_1^{i_1}$ ถ้า $p_2^{i_2}...p_k^{i_k} \geqslant 3$ จะพบว่า $n!$ มี $p_1^{i_1}$ ที่หาร n ลงตัวอยู่อย่างน้อย 3 ตัว

ซึ่งจาก (1) ถ้าไม่รวม n จะมี$p_1^{i_1}$ ที่หาร n ลงตัวอยู่อย่างน้อย 2 ตัว ทำให้ $(p_1^{2i_1}) \ | \ (n-2)!$

ในทำนองเดียวกันกับ $p_2,...,p_k$

ดังนั้น จะมี $p_m, 1 \leqslant m \leqslant k$ ซึ่ง $p_1^{i_1}...p_{m-1}^{i_{m-1}}p_{m+1}^{i_{m+1}}...p_k^{i_k} < 3$ หรือจะสามารถเขียน $n = p_1^{i_1}$

$\therefore n = p_1^{i_1} \vee n = 2p_1^{i_1}$ ซึ่งแทนค่ากลับพบว่าจริง
พิจารณากรณี $n = 2,3$ พบว่าเป็นจริง

ดังนั้น $ n = p^{i} \vee n = 2p^{i}$ เมื่อ p เป็นจำนวนเฉพาะ, i เป็นจำนวนนับ

บรรทัดนี้อ่ะครับ ตอนแรกเข้าใจ(พิมพ์ผิดนะครับ) แต่ทำไมถึงสรุปได้อย่างหลังอ่ะครับ
__________________
...สีชมพูจะไม่จางด้วยเหงื่อ แต่จะจางด้วยนํ้าลาย...

05 พฤษภาคม 2012 19:39 : ข้อความนี้ถูกแก้ไขแล้ว 2 ครั้ง, ครั้งล่าสุดโดยคุณ ~ArT_Ty~
ตอบพร้อมอ้างอิงข้อความนี้
  #389  
Old 05 พฤษภาคม 2012, 19:58
AnDroMeDa's Avatar
AnDroMeDa AnDroMeDa ไม่อยู่ในระบบ
ลมปราณบริสุทธิ์
 
วันที่สมัครสมาชิก: 10 ตุลาคม 2011
ข้อความ: 114
AnDroMeDa is on a distinguished road
Default

อ้างอิง:
ข้อความเดิมเขียนโดยคุณ Beatmania View Post
ถ้า $xyz=1$ และ $x,y,z \in \unicode{8477} $ แล้วจงแสดงว่า

$\frac{x^2}{(x-1)^2} +\frac{y^2}{(y-1)^2} +\frac{z^2}{(z-1)^2} \geqslant 1$


จงแสดงว่า ทุกจำนวนเต็มบวก $n$ มีจำนวนนับ $n$ จำนวนที่ถัดกัน โดยไม่มีจำนวนใดอยู่ในรูป $p^k$ สำหรับจำนวนเฉพาะ p บางจำนวน
ข้อแรกจัดรูปโหดๆได้ $$\frac{x^2}{(x-1)^2} +\frac{y^2}{(y-1)^2} +\frac{z^2}{(z-1)^2}-1 \geqslant 0\Leftrightarrow (\frac{x}{x-1} +\frac{y}{y-1} +\frac{z}{z-1} -1)^2\geqslant 0 $$
ซึ่งอสมการเป็นสมการก็ต่อเมื่อ $\frac{x}{x-1} +\frac{y}{y-1} +\frac{z}{z-1}=1\Leftrightarrow \frac{1}{x} +\frac{1}{y} +\frac{1}{z} =3$จาก$xyz=1$ จะได้ว่ามีจำนวนตรรกยะ $x,y,z$ เป็นอนันต์ที่ทำให้เกิดสมการ

ข้อสองนี่หมายความว่าไรรึครับ

อ้างอิง:
ข้อความเดิมเขียนโดยคุณ ~ArT_Ty~ View Post
มีฟังก์ชัน $f:\mathbb{N} \rightarrow \mathbb{N} $ ที่สอดคล้องกับ $f(f(n))=2n$ ทุก $n$ ที่เป็นจำนวนนับหรือไม่
จะตอบอย่างนี้ได้มั้ยครับ $f(i)=j,f(j)=2i,\forall i,j\in \mathbb{N} ,f(k)=2f(\frac{k}{2} )$เมื่อ $k$ เป็นจำนวนคู่ จะได้สมการเชิงฟังก์ชันที่สอดคล้องกับโจทย์กำหนด
ตอบพร้อมอ้างอิงข้อความนี้
  #390  
Old 05 พฤษภาคม 2012, 20:01
AnDroMeDa's Avatar
AnDroMeDa AnDroMeDa ไม่อยู่ในระบบ
ลมปราณบริสุทธิ์
 
วันที่สมัครสมาชิก: 10 ตุลาคม 2011
ข้อความ: 114
AnDroMeDa is on a distinguished road
Default

อ้างอิง:
ข้อความเดิมเขียนโดยคุณ Thgx0312555 View Post
(#378)::

พิจารณา $n \geqslant 4$

จะพบว่า $\dfrac{n}{2}$ $\leqslant$ $n-2$ นั่นคือตัวประกอบทั้งหมดที่ไม่ใช่ตัวมันเองจะมีค่าน้อยกว่าหรือเท่ากับ n-2 ---(1)

ให้ $n = p_1^{i_1}p_2^{i_2}...p_k^{i_k}$ พิจารณา $p_1^{i_1}$ ถ้า $p_2^{i_2}...p_k^{i_k} \geqslant 3$ จะพบว่า $n!$ มี $p_1^{i_1}$ ที่หาร n ลงตัวอยู่อย่างน้อย 3 ตัว

ซึ่งจาก (1) ถ้าไม่รวม n จะมี$p_1^{i_1}$ ที่หาร n ลงตัวอยู่อย่างน้อย 2 ตัว ทำให้ $(p_1^{2i_1}) \ | \ (n-2)!$

ในทำนองเดียวกันกับ $p_2,...,p_k$

ดังนั้น จะมี $p_m, 1 \leqslant m \leqslant k$ ซึ่ง $p_1^{i_1}...p_{m-1}^{i_{m-1}}p_{m+1}^{i_{m+1}}...p_k^{i_k} < 3$ หรือจะสามารถเขียน $n = p_1^{i_1}$

$\therefore n = p_1^{i_1} \vee n = 2p_1^{i_1}$ ซึ่งแทนค่ากลับพบว่าจริง
พิจารณากรณี $n = 2,3$ พบว่าเป็นจริง

ดังนั้น $ n = p^{i} \vee n = 2p^{i}$ เมื่อ p เป็นจำนวนเฉพาะ, i เป็นจำนวนนับ
ตรงสีแดงยังทำไม่เสร็จทำให้คำตอบเกินนะครับ
ตอบพร้อมอ้างอิงข้อความนี้
ตั้งหัวข้อใหม่ Reply



กฎการส่งข้อความ
คุณ ไม่สามารถ ตั้งหัวข้อใหม่ได้
คุณ ไม่สามารถ ตอบหัวข้อได้
คุณ ไม่สามารถ แนบไฟล์และเอกสารได้
คุณ ไม่สามารถ แก้ไขข้อความของคุณเองได้

vB code is On
Smilies are On
[IMG] code is On
HTML code is Off
ทางลัดสู่ห้อง


เวลาที่แสดงทั้งหมด เป็นเวลาที่ประเทศไทย (GMT +7) ขณะนี้เป็นเวลา 11:49


Powered by vBulletin® Copyright ©2000 - 2024, Jelsoft Enterprises Ltd.
Modified by Jetsada Karnpracha